Akademisyenler öncülüğünde matematik/fizik/bilgisayar bilimleri soru cevap platformu
0 beğenilme 0 beğenilmeme
1.5k kez görüntülendi

1-) $e^{2z}+e^{z}+1=0$

2-) $e^{\dfrac {1} {z}}=-1$

3-) $e^{z-1}=-ie^3$

Denklemini sağlayan karmaşık sayı değerlerini bulunuz.

Konuyu anlamakta biraz zorlanıyorum. Soruları biraz detaylı çözerseniz sevinirim. Şimdiden teşekkürler :)

Lisans Matematik kategorisinde (20 puan) tarafından 
tarafından düzenlendi | 1.5k kez görüntülendi

1 cevap

0 beğenilme 0 beğenilmeme

1) $e^z=u$ dersek $u^2+u+1=0$ gelir.

2) $e^{\pi i}=-1$ o halde $\frac{1}{z}=i\pi+2k\pi$ olmali, $k$ tam sayi

3) $e^{z-2}=-i$ ve $-i=e^{\frac{3\pi}{2}+2k\pi}$

(25.3k puan) tarafından 
$2k\pi+i\pi \rightarrow 2k\pi - i \pi$

k orda değişen bir sayı hocam :) k nın karesi alıyoz orda. Değer değişmez mi?


Birde diğer sorularda biz karmaşık analiz görürken hocam öyle göstermedi. ln'nini alıyoz her iki tarafın. Ordan bir elementar fonksiyon kuralı var. Ordan öyle çıkarıyoz. Benim yaptığımda doğru değil mi hocam?

$a+bi$ yazdigimizda $a,b$ de degisken ama eslenigine $a-bi$ diyoruz.

Hocam diğer sorularda benim cevaplarım doğru mu peki?

geriye $3$ kaliyor, onda da cevaplarimiz farkli zaten.araya bir de $x$ koymussun?

Hocam ben fazladan oraya ln yazdım. O gereksiz mi?

Reel sayilarda: $2^x=2^y$ ise $x=y$ deriz ama $e^x=a$ ise $x=\ln a$ deriz..

$e^{i\theta_1}=e^{i\theta_2}$ ise $\theta_2-\theta_1=2k\pi$'dir.

yani benim oraya ln i koymam doğru mu?

evet..              

$e^{i\theta}=\cos\theta+i\sin\theta$

Hocam bakın bendeki formülde şu: $\ln z=(log_{e}\left| z\right| +i\arg \left( z\right) $ yani $\ln r+i\arg \left( z\right) $

Bu soru lise sorusu mu universite sorusu mu?

Üniversite sorusudur.

O zaman bu iki formulun birbirine esit mi yoksa formullerden birisi yanlis mi bulmak soruyu cozenin gorevi. Hatta o formuller neden var, nereden gelmisler, ispatlamak da.

Tabii lise sorusu olsaydi. liseli arkadaslarin da ayni seyi yapmaya calismalari dogrusu. Ama universiteli icin dogrudan ziyade mecburiyet.


özelden mesaj attım bakarmısınız?

Hangi formulleri?

Biraz yukarda hocam. Araya baya yazışma girdi.


$\ln z=(log_{e}\left| z\right| +i\arg \left( z\right) $ yani $\ln r+i\arg \left( z\right) $

Daha önceden bununla ilgili bir soru çözmüştüm. Sitede mevcut da, soru neydi, nerdedir bilmiyorum.

20,200 soru
21,726 cevap
73,275 yorum
1,887,773 kullanıcı